中国高考数学压轴题

\section{中国高考数学压轴题}

 


\begin{example}
(2009年湖南)
\end{example}
\begin{proof}
解: (I)设满足题设的等比数列为$\{a_n\}$,则$a_n=q^{n-1}$.

于是$|a_n-a_{n-1}|=|q^{n-1}-q^{n-2}|=|q|^{n-2}|q-1|,n\geqslant 2$.

因此
\begin{align*}
&\left| a_{n+1}-a_n \right|+\left| a_n-a_{n-1} \right|+\cdots +\left| a_2-a_1 \right|
\\
&=\left| q-1 \right|\left( 1+\left| q \right|+\left| q \right|+\cdots +\left| q \right|^{n-1} \right).
\end{align*}

因为$|q|<1$,所以
$$
1+\left| q \right|+\left| q \right|+\cdots +\left| q \right|^{n-1}=\frac{1-\left| q \right|^n}{1-\left| q \right|}<\frac{1}{1-\left| q \right|}.
$$

$$
\left| a_{n+1}-a_n \right|+\left| a_n-a_{n-1} \right|+\cdots +\left| a_2-a_1 \right|<\frac{\left| q-1 \right|}{1-\left| q \right|}.
$$
故首项为$1$,公比为$q\, (|q|<1)$的等比数列是$B$—数列.

(II)命题1:若数列$\{x_n\}$是$B$—数列,则数列$\{S_n\}$是$B$—数列.

此命题为假命题.

事实上,设$x_n=1,n\in \mathbb{N}^\ast$,易知数列$\{x_n\}$是$B$—数列.

但$S_n=n,\left| S_{n+1}-S_n \right|+\left| S_n-S_{n-1} \right|+\cdots +\left| S_2-S_1 \right|=n$.由$n$的任意性知,数列$\{S_n\}$不是$B$—数列.

命题2:若数列$\{S_n\}$是$B$—数列,则数列$\{x_n\}$是$B$—数列.

此命题为真命题.

事实上,因为数列$\{S_n\}$是$B$—数列,所以存在正数$M$,对任意的$n\in \mathbb{N}^\ast$,

$$
\left| S_{n+1}-S_n \right|+\left| S_n-S_{n-1} \right|+\cdots +\left| S_2-S_1 \right|\leqslant M,
$$

$$
\left| x_{n+1}\right|+\left|x_n\right|+\cdots +\left| x_2\right|\leqslant M.
$$
于是
\begin{align*}
&\left| x_{n+1}-x_n \right|+\left| x_n-x_{n-1} \right|+\cdots +\left| x_2-x_1 \right|
\\
&\leqslant \left| x_{n+1} \right|+2\left| x_n \right|+2\left| x_{n-1} \right|+\cdots +2\left| x_2 \right|+\left| x_1 \right|
\\
&\leqslant 2M+\left| x_1 \right|.
\end{align*}
所以数列$\{x_n\}$是$B$—数列.

(注:按题中要求组成其他命题解答时,仿上述解法)

(III)若数列$\{a_n\},\{b_n\}$是$B$—数列,则存在正数$M_1,M_2$,对任意的$n\in \mathbb{N}^\ast$,有
\begin{align*}
\left| a_{n+1}-a_n \right|+\left| a_n-a_{n-1} \right|+\cdots +\left| a_2-a_1 \right| &\leqslant M_1;
\\
\left| b_{n+1}-b_n \right|+\left| b_n-b_{n-1} \right|+\cdots +\left| b_2-b_1 \right| &\leqslant M_2.
\end{align*}
注意到
\begin{align*}
\left| a_n \right| &=\left| a_n-a_{n-1}+a_{n-1}-a_{n-2}+\cdots +a_2-a_1+a_1 \right|\leqslant M_1
\\
&\leqslant \left| a_{n+1}-a_n \right|+\left| a_n-a_{n-1} \right|+\cdots +\left| a_2-a_1 \right|+\left| a_1 \right|
\\
&\leqslant M_1+\left| a_1 \right|,
\end{align*}
同理, $\left| b_n \right|\leqslant M_2+\left| b_1 \right|$.

记$K_1=M_1+|a_1|,K_2=M_2+|b_1|$,则有
\begin{align*}
\left| a_{n+1}b_{n+1}-a_nb_n \right| &=\left| a_{n+1}b_{n+1}-a_nb_{n+1}+a_nb_{n+1}-a_nb_n \right|
\\
&\leqslant \left| b_{n+1} \right|\left| a_{n+1}-a_n \right|+\left| a_n \right|\left| b_{n+1}-b_n \right|
\\
&\leqslant K_2\left| a_{n+1}-a_n \right|+K_1\left| b_{n+1}-b_n \right|.
\end{align*}

因此
\begin{align*}
&\left| a_{n+1}b_{n+1}-a_nb_n \right|+\left| a_nb_n-a_{n-1}b_{n-1} \right|+\cdots +\left| a_2b_2-a_1b_1 \right|
\\
&\leqslant K_2\left( \left| a_{n+1}-a_n \right|+\left| a_n-a_{n-1} \right|+\cdots +\left| a_2-a_1 \right| \right)
\\
&\quad +K_1\left( \left| b_{n+1}-b_n \right|+\left| b_n-b_{n-1} \right|+\cdots +\left| b_2-b_1 \right| \right)
\\
&\leqslant K_2M_1+K_1M_2.
\end{align*}
故数列$\{a_nb_n\}$是$B$—数列.
\end{proof}


\begin{example}
(1979年高考,潘承彪)叙述并证明勾股定理.
\end{example}
\begin{proof}

\end{proof}

\begin{example}
(1982年高考)抛物线$y^2=2px$的内接三角形有两边与抛物线$x^2=2qy$相切,证明这个三角形的第三边也与$x^2=2qy$相切.
\end{example}
\begin{proof}
不失一般性,设$p>0,q>0$.又设$y^2=2px$的内接三角形顶点为$A_1(x_1,y_1),A_2(x_2,y_2),A_3(x_3,y_3)$.因此$y_1^2=2px_1,y_2^2=2px_2,y_3^2=2px_3$.

其中$y_1\neq y_2,y_2\neq y_3,y_3\neq y_1$.

依题意,设$A_1A_2,A_2A_3$与抛物线$x^2=2qy$相切,要证$A_3A_1$也与抛物线$x^2=2qy$相切.

因为$x^2=2qy$在原点$O$处的切线是$y^2=2px$的对称轴,所以原点$O$不能是所设内接三角形的顶点.即$(x_1,y_1),(x_2,y_2),(x_3,y_3)$,都不能是$(0,0)$;
又因$A_1A_2$与$x^2=2qy$相切,所以$A_1A_2$不能与$y$轴平行,即$x_1\neq x_2,y_1\neq -y_2$,直线$A_1A_2$的方程是
$$y-y_1=\frac{y_2-y_1}{x_2-x_1}(x-x_1),$$

因为$y_2^2-y_1^2=(y_2-y_1)(y_2+y_1)=2p(x_2-x_1)$,所以$A_1A_2$的方程是$y=\frac{2p}{y_1+y_2}x+\frac{y_1y_2}{y_1+y_2}$.

$A_1A_2$与抛物线$x^2=2qy$交点的横坐标满足
$$x^2-\frac{4pq} {y_1+y_2}x-\frac{2qy_1y_2}{y_1+y_2}=0,$$
由于$A_1A_2$与抛物线$x^2=2qy$相切,上面二次方程的判别式
$$
\Delta =\left( -\frac{4pq}{y_1+y_2} \right) ^2+4\left( \frac{2qy_1y_2}{y_1+y_2} \right) =0.
$$
化简得
\[2p^2q+y_1y_2(y_1+y_2)=0.\tag*{\ding{172}}\]

同理由于$A_2A_3$与抛物线$x^2=2qy$相切, $A_2A_3$也不能与$y$轴平行,即$x_2\neq x_3,y_2\neq -y_3$.

同样得到
\[2p^2q+y_2y_3(y_2+y_3)=0.\tag*{\ding{173}}\]
由\ding{172}\ding{173}两方程及$y_2\neq 0,y_1\neq y_3$,得$y_1+y_2+y_3=0$.

由上式及$y_2\neq 0$,得$y_3\neq -y_1$,也就是$A_3A_1$也不能与$y$轴平行.

今将$y_2=-y_1-y_3$代人\ding{172}式得:
\[2p^2q+y_3y_1(y_3+y_1)=0.\tag*{\ding{174}}\]
\ding{174}式说明$A_3A_1$与抛物线$x^2=2qy$的两个交点重合,即$A_3A_1$与抛物线$x^2 =2qy$相切.

所以只要$A_1A_2,A_2A_3$与抛物线$x^2=2qy$相切,则$A_3A_1$必与抛物线$x^2=2qy$相切.
\end{proof}

\begin{example}
(1983年高考) 1.已知$a,b$为实数,并且$e<a<b$,其中$e$是自然对数的底,证明$a^b>b^a$.

2.如果正实数$a,b$满足$a^b=b^a$,且$a<1$,证明$a=b$.
\end{example}
\begin{proof}

\end{proof}


\begin{example}
(1984年高考) 设$a>2$,给定数列$\{x_n\}$,其中$x_1=a,x_{n+1}=\frac{x_n^2}{2(x_n-1)}\,(n=1,2,\cdots)$.

求证:

1. $x_n>2$,且$\frac{x_{n+1}}{x_n}<1\, (n=1,2,\cdots)$;

2.如果$a\leqslant 3$,那么$x_n\leqslant 2+\frac{1}{2^{n-1}}\, (n=1,2,\cdots)$;


3.如果$a>3$,那么当$n\geqslant \frac{\lg\frac{a}{3}}{\lg\frac{4}{3}}$时,必有$x_{n+1}<3$.
\end{example}
\begin{proof}

\end{proof}


\begin{example}
(1985年高考)设$a_n=\sqrt{1\cdot 2}+\sqrt{2\cdot 3}+\cdots+ \sqrt{n(n+1)}\,(n=1,2,\cdots)$.

(I)证明不等式$\frac{n(n+1)}{2}<a_n<\frac{(n+1)^2}{2}$对所有的正整数$n$都成立.

(II)设$b_n=\frac{a_n}{n(n+1)}\, (n=1,2,\cdots)$,用极限定义证明$\lim_{n\to\infty}b_n =\frac{1}{2}$.
\end{example}
\begin{proof}

\end{proof}


\begin{example}
(1986年高考)已知$x_1>0,x\neq 1$,且$x_{n+1}=\frac{x_n(x_n^2+3)}{3x_n^2+1}\,
(n=1,2,\cdots)$.试证:数列$\{x_n\}$或者对任意自然数$n$都满足$x_n<x_{n+1}$,或者对任意自然数$n$都满足$x_n>x_{n+1}$.
\end{example}
\begin{proof}

\end{proof}

 

\begin{example}
(1987年高考)求$\sin10^\circ\sin30^\circ\sin50^\circ\sin70^\circ$的值.
\end{example}
\begin{proof}

\end{proof}

\begin{example}
(1987年高考)设数列$a_1,a_2,\cdots,a_n,\cdots$的前$n$项的和$S_n$与$a_n$的关系是$S_n=-ba_n+1-\frac{1}{(1+b)^n}$,其中$b$是与$n$无关的常数,且$b\neq -1$.

(1)求$a_n$和$a_{n-1}$的关系式;

(2)写出用$n$和$b$表示$a_n$的表达式;

(3)当$0<b<1$时,求极限$\lim_{n\to\infty}S_n$.
\end{example}
\begin{proof}

\end{proof}


\begin{example}
(1987年高考)定长为$3$的线段$AB$的两个端点在抛物线$y^2=x$上移动,记线段$AB$的中点为$M$.求点$M$到$y$轴的最短距离,并求此时点$M$的坐标.
\end{example}
\begin{proof}

\end{proof}


\begin{example}
(1996年高考)已知$a,b,c$是实数,函数$f(x)=ax^2+bx+c,g(x)=ax+b$,当$-1\leqslant x\leqslant 1$时, $|f(x)|\leqslant 1$.

(I)证明: $|c|\leqslant 1$;

(II)证明:当$-1\leqslant x\leqslant 1$时, $|g(x)|\leqslant 2$;

(III)设$a>0$,当$-1\leqslant x\leqslant 1$时, $g(x)$的最大值为$2$,求$f(x)$.
\end{example}
\begin{proof}

\end{proof}

\begin{example}
(1997年高考)设二次函数$f(x)=ax^2+bx+c\,(a>0)$,方程$f(x)-x=0$的两个根$x_1,x_2$满足$0<x_1<x_2<\frac{1}{a}$.

(I)当$x\in (0,x_1)$时,证明$x<f(x)<x_1$;

(II)设函数$f(x)$的图象关于直线$x=x_0$对称,证明$x_0<\frac{x_1}{2}$.
\end{example}
\begin{proof}

\end{proof}

2005chongqingtuianjing

\begin{example}
(2001年全国旧课程高考)已知$i,m,n$是正整数,且$1<i\leqslant m<n$.

(I)证明$n^iA_m^i<m^iA_n^i$;

(II)证明$(1+m)^n>(1+n)^m$.
\end{example}
\begin{proof}

\end{proof}


\begin{example}
(2001年全国旧课程高考)设$f(x)$是定义在$\mathbb{R}$上的偶函数,其图象关于直线$x=1$对称,对任意到$x_1,x_2\in \left[0,\frac{1}{2}\right]$,都有$f(x_1+x_2)=
f(x_1)f(x_2)$,且$f(1)=a>0$.

(I)求$f\left(\frac{1}{2}\right)$及
$f\left(\frac{1}{4}\right)$;

(II)证明$f(x)$是周期函数;

(III)记$a_n= f\left(2n+\frac{1}{2n}\right)$,
求$\lim_{n\to\infty} (\ln a_n)$.
\end{example}
\begin{proof}

\end{proof}

\begin{example}
(2002年全国旧课程高考)设数列$\{a_n\}$满足: $a_{n+1}= a_n^2-nan+1,n=1,2,3,\cdots$.

(I)当$a_1=2$时,求$a_2,a_3,a_4$,并由此猜测出$a_n$的一个通项公式;

(II)当$a_1\geqslant 3$时,证明对所有的$n\geqslant 1$,有

(i) $a_n\geqslant n+2$;

(ii) $\frac{1}{1+a_1}+\frac{1}{1+a_2}
+\cdots+\frac{1}{1+a_n}\leqslant\frac{1}{2}$.
\end{example}
\begin{proof}

\end{proof}

 

\begin{example}
(2002年全国新课程高考)已知$a>0$,函数$f(x)=\frac{1-ax}{x},x\in (0,+\infty)$.设$0<x_1<\frac{2}{a}$,记曲线$y=f(x)$在点$M(x_1,f(x_1))$处的切线为$l$.

(I)求$l$的方程;

(II)设$l$与$x$轴交点为$(x_2,0)$.证明:

(i) $0<x_2\leqslant \frac{1}{a}$;

(i)若$x_1<\frac{1}{a}$,则$x_1<x_2<\frac{1}{a}$.
\end{example}
\begin{proof}

\end{proof}


\begin{example}
(2002年广东高考)已知$a>0$,函数$f(x)=ax-bx^2$.

(I)当$b>0$时,若对任意$x\in \mathbb{R}$都有$f(x)\leqslant 1$,证明$a\leqslant 2\sqrt{b}$;

(II)当$b>1$时,证明:对任意$x\in [0,1]$, $|f(x)|\leqslant 1$的充要条件是$b-1\leqslant a\leqslant 2\sqrt{b}$;

(III)当$0<b\leqslant 1$时,讨论:对任意$x\in [0,1]$, $|f(x)|\leqslant 1$的充要条件.
\end{example}
\begin{proof}

\end{proof}

 

\begin{example}
(2005年重庆)数列$\{a_n\}$满足$a_1=1$且$a_{n+1}=\left(1+\frac{1}{n^2+n}\right)a_n+\frac{1}{2^n}\, (n\geqslant 1)$.

(I)用数学归纳法证明: $a_n\geqslant 2\,(n\geqslant 2)$;

(II)已知不等式$\ln (1+x)<x$对$x>0$成立,证明: $a_n <e^2\,(n\geqslant 1)$,其中无理数$e= 2.718\,28\cdots$.
\end{example}
\begin{proof}

\end{proof}

 

\begin{example}
(2005年湖北)已知不等式$\frac{1}{2}+\frac{1}{3}+\cdots +\frac{1}{n}>\frac{1}{2}[\log_2n]$,其中$n$为大于$2$的整数, $[\log_2n]$表示不超过$\log_2n$的最大整数.

设数列$\{a_n\}$的各项为正,且满足
$$a_1=b\,(b>0),a_n\leqslant\frac{na_{n-1}}{n+a_{n-1}},
n=2,3,4,\cdots.$$

(I)证明$a_n<\frac{2b}{2+b(\log_2n)}$, $n=3,4,5,\cdots$.

(II)猜测:数列$\{a_n\}$是否有极限?如果有,写出极限的值(不必证明).

(III)试确定一个正整数$N$,使得当$n>N$时,对任意$b>0$,都有$a_n<\frac{1}{5}$.
\end{example}
\begin{proof}

\end{proof}

 

\begin{example}
(2005年湖南)已知函数$f(x)=\ln x,g(x)=\frac{1}{2}ax^2+bx,a\neq 0$.

(I)若$b=2$,且函数$h(x)=f(x)-g(x)$存在单调递减区间,求$a$的取值范围.

(II)设函数$f(x)$的图象$C_1$与函数$g(x)$的图象$C_2$交于点$P,Q$,过线段$PQ$的中点作$x$轴的垂线分别交$C_1,C_2$于点$M,N$.证明$C_1$在点$M$处的切线与$C_2$在点$N$处的切线不平行.
\end{example}
\begin{proof}

\end{proof}

 


\begin{example}
(2005年天津)设函数$f(x)=x\sin x\,(x\in \mathbb{R})$.

(I)证明$f(x+2k\pi)-f(x)=2k\pi\sin x$,其中$k$为整数;

(II)设$x_0$为$f(x)$的一个极值点,证明$[f(x_0)]^2=\frac{x_0^4}{1+x_0^2}$;

(III)设$f(x)$在$(0,+\infty)$内的全部极值点按从小到大的顺序排列为$a_1,a_2,\cdots,a_n,\cdots$,证明$\frac{\pi}{2}<a_{n+1}-a_n<\pi\,(n=1,2,\cdots)$.
\end{example}
\begin{proof}

\end{proof}


\begin{example}
(2003高考江苏卷压轴题,缺图)设$a>0$,如图,已知直线$l:y=ax$及曲线$C:y=x^2$, $C$上的点$Q_1$的横坐标为$a_1\,(0<a_1<a)$.从$C$上的点$Q_n\,(n\geqslant 1)$作直线平行于$x$轴,交直线$l$于点$P_{n+1}$,再从点$P_{n+1}$作直线平行于$y$轴,交曲线$C$于点$Q_{n+1}$. $Q_n\,(n=1,2,3,\cdots)$的横坐标构成数列$\{a_n\}$.
\begin{enumerate}
\item[(I)] 试求$a_{n+1}$与$a_n$的关系,并求$\{a_n\}$的通项公式;
\item[(II)] 当$a=1,a_1\leqslant \frac{1}{2}$时,证明$\sum_{k=1}^{n}(a_k-a_{k+1})a_{k+2}<\frac{1}{32}$;

\item[(III)] 当$a=1$时,证明$\sum_ {k=1}^{n}(a_k-a_{k+1})a_{k+2}<\frac{1}{3}$.
\end{enumerate}
\end{example}
\begin{solution}%472
(I)因为$Q_n\left( a_n,a_{n}^{2} \right) ,P_{n+1}\left( \frac{1}{a}\cdot a_{n}^{2},a_{n}^{2} \right) ,Q_{n+1}\left( \frac{1}{a}\cdot a_{n}^{2},\frac{1}{a^2}a_{n}^{4} \right)$,
所以$a_{n+1}=\frac{1}{a}\cdot a_{n}^{2}$,则
\begin{align*}
a_n &=\frac{1}{a}\cdot a_{n-1}^{2}=\frac{1}{a}\left( \frac{1}{a}\cdot a_{n-2}^{2} \right) ^2=\left( \frac{1}{a} \right) ^{1+2}\cdot a_{n-2}^{2^2}
\\
&=\left( \frac{1}{a} \right) ^{1+2}\left( \frac{1}{a}\cdot a_{n-3}^{2} \right) ^{2^2}=\left( \frac{1}{a} \right) ^{1+2+2^2}a_{n-3}^{2^3}
\\
&=\cdots
\\
&=\left( \frac{1}{a} \right) ^{1+2+\cdots +2^{n-2}}a_{1}^{2^{n-1}}=\left( \frac{1}{a} \right) ^{2^{n-1}-1}a_{1}^{2^{n-1}}=a\left( \frac{a_1}{a} \right) ^{2^{n-1}},
\end{align*}
所以$a_n=a\left( \frac{a_1}{a} \right) ^{2^{n-1}}$.

(II)证明:由$a=1$知$a_{n+1}=a_n^2$.

因为$a_1\leqslant \frac{1}{2}$,所以$a_2\leqslant \frac{1}{4},a_3\leqslant \frac{1}{16}$.

因为当$k\geqslant 1$时, $a_{k+2}\leqslant a_3\leqslant \frac{1}{16}$.所以
\begin{align*}
\sum_{k=1}^n{\left( a_k-a_{k+1} \right) a_{k+2}} &\leqslant \frac{1}{16}\sum_{k=1}^n{\left( a_k-a_{k+1} \right)}
\\
&=\frac{1}{16}\left( a_1-a_{n+1} \right) <\frac{1}{32}.
\end{align*}

(III)证明:由(I)知,当$a=1$时, $a_n=a_1^{2^{n-1}}$.因此
\begin{align*}
\sum_{k=1}^n{\left( a_k-a_{k+1} \right) a_{k+2}} &=\sum_{k=1}^n{\left( a_{1}^{2^{k-1}}-a_{1}^{2^k} \right) a_{1}^{2^{k+1}}}\leqslant \sum_{i=1}^{2^n-1}{\left( a_{1}^{i}-a_{1}^{i+1} \right) a_{1}^{2i+2}}
\\
&=\left( 1-a_1 \right) a_{1}^{2}\sum_{i=1}^{2^n-1}{a_{1}^{3i}}<\left( 1-a_1 \right) a_{1}^{2}\cdot \frac{a_{1}^{3}}{1-a_{1}^{3}}
\\
&=\frac{a_{1}^{5}}{1+a_1+a_{1}^{2}}<\frac{1}{3}.
\end{align*}
\end{solution}

\begin{example}
(2010年全国高中数学联赛江苏赛区复赛,缺图)数列$\{a_n\}$中,已知$a_1\in (1,2),a_{n+1}=a_n^3-3a_n^2+3a_n,n\in \mathbb{N}^\ast$,求证:
$$
\left( a_1-a_2 \right) \left( a_3-1 \right) +\left( a_2-a_3 \right) \left( a_4-1 \right) +\cdots +\left( a_n-a_{n+1} \right) \left( a_{n+2}-1 \right) <\frac{1}{4}.
$$
\end{example}
\begin{proof}
\textbf{方法一.}由$a_{n+1}=a_n^3-3a_n^2+3a_n$,得$a_{n+1}-1=(a_n-1)^3$.

令$b_n=a_n-1$,则$0<b_1<1$, $b_{n+1}=b_n^3<b_n$, $0<b_n<1$.

所以
\begin{align*}
\left( a_k-a_{k+1} \right) \left( a_{k+2}-1 \right) &=\left( b_k-b_{k+1} \right) b_{k+2}=\left( b_k-b_{k+1} \right) b_{k+1}^{3}
\\
&<\frac{1}{4}\left( b_k-b_{k+1} \right) \left( b_{k}^{3}+b_{k}^{2}b_{k+1}+b_kb_{k+1}^{2}+b_{k+1}^{3} \right) =\frac{1}{4}\left( b_{k}^{4}-b_{k+1}^{4} \right).
\end{align*}
所以
\begin{align*}
&\left( a_1-a_2 \right) \left( a_3-1 \right) +\left( a_2-a_3 \right) \left( a_4-1 \right) +\cdots +\left( a_n-a_{n+1} \right) \left( a_{n+2}-1 \right)
\\
&<\frac{1}{4}\left( b_{1}^{4}-b_{2}^{4} \right) +\frac{1}{4}\left( b_{2}^{4}-b_{3}^{4} \right) +\cdots +\frac{1}{4}\left( b_{n}^{4}-b_{n+1}^{4} \right)
\\
&=\frac{1}{4}\left( b_{1}^{4}-b_{n+1}^{4} \right) <\frac{1}{4}b_{1}^{4}<\frac{1}{4}.
\end{align*}

\textbf{方法二.}由$a_{n+1}=a_n^3-3a_n^2+3a_n$,得$a_{n+1}-1=(a_n-1)^3$.

令$b_n=a_n-1$,则$0<b_1<1$, $b_{n+1}=b_n^3$, $0<b_n<1$.

所以
\begin{align*}
&\left( a_1-a_2 \right) \left( a_3-1 \right) +\left( a_2-a_3 \right) \left( a_4-1 \right) +\cdots +\left( a_n-a_{n+1} \right) \left( a_{n+2}-1 \right)
\\
&=\left( b_1-b_2 \right) b_3+\left( b_2-b_3 \right) b_4+\cdots +\left( b_n-b_{n+1} \right) b_{n+2}
\\
&=\left( b_1-b_2 \right) b_{2}^{3}+\left( b_2-b_3 \right) b_{3}^{3}+\cdots +\left( b_n-b_{n+1} \right) b_{n+1}^{3}
\\
&<\int_0^1{x^3dx}=\frac{1}{4}.
\end{align*}
\end{proof}
%https://wenku.baidu.com/view/25251cae28ea81c759f57818.html
2003年北京

\begin{example}
(2004年江苏)已知函数$f(x)\, (x\in\mathbb{R})$满足下列条件:对任意的实数$x_1,x_2$都有
\[\lambda (x_1-x_2)^2\leq (x_1-x_2)[f(x_1)-f(x_2)]\]

\[|f(x_1)-f(x_2)|\leq |x_1-x_2|,\]
其中$\lambda$是大于$0$的常数.设实数$a_0,a,b$满足$f(a_0)=0$和$b=a-\lambda f(a)$.
\begin{enumerate}
\item[(I)] 证明$\lambda\leq 1$,并且不存在$b_0\neq a_0$,使得$f(b_0)=0$;

\item[(II)] 证明$(b-a_0)^2\leq (1-\lambda^2)(a-a_0)^2$;

\item[(III)] 证明: $[f(b)]^2\leq (1-\lambda^2)[f(a)]^2$.
\end{enumerate}
\end{example}
\begin{proof}
(I)任取$x_1,x_2\in \mathbb{R},x_1\neq x_2$,则由
\[
\lambda \left( x_1-x_2 \right) ^2\leqslant \left( x_1-x_2 \right) \left[ f\left( x_1 \right) -f\left( x_2 \right) \right] \tag*{\ding{172}}
\]

\[
\left| f\left( x_1 \right) -f\left( x_2 \right) \right|\leqslant \left| x_1-x_2 \right| \tag*{\ding{173}}
\]
可知
\begin{align*}
\left( x_1-x_2 \right) ^2 &\leqslant \left( x_1-x_2 \right) \left[ f\left( x_1 \right) -f\left( x_2 \right) \right]
\\
&\leqslant \left| x_1-x_2 \right|\left| f\left( x_1 \right) -f\left( x_2 \right) \right|\leqslant \left| x_1-x_2 \right|^2,
\end{align*}
从而$\lambda\leqslant 1$.

假设有$b_0\neq a_0$,使得$f(b_0)=0$.则由\ding{172}式知
$$
0<\lambda \left( a_0-b_0 \right) ^2\leqslant \left( a_0-b_0 \right) \left[ f\left( a_0 \right) -f\left( b_0 \right) \right] =0,
$$
矛盾.所以不存在$b_0\neq a_0$,使得$f(b_0)=0$.

(II)由
\[b=a-\lambda f(a),\tag*{\ding{174}}\]
可知
\[
\left( b-a_0 \right) ^2=\left( a-a_0-\lambda f\left( a \right) \right) ^2=\left( a-a_0 \right) ^2-2\lambda \left( a-a_0 \right) f\left( a \right) +\lambda ^2\left[ f\left( a \right) \right] ^2.\tag*{\ding{175}}
\]
由$f(a_0)=0$和\ding{172}式,得
\[
\left( a-a_0 \right) f\left( a \right) =\left( a-a_0 \right) \left[ f\left( a \right) -f\left( a_0 \right) \right] \geqslant \lambda \left( a-a_0 \right) ^2.\tag*{\ding{176}}
\]
由$f(a_0)=0$和\ding{173}式知,
\[
\left[ f\left( a \right) \right] ^2=\left[ f\left( a \right) -f\left( a_0 \right) \right] ^2\leqslant \left( a-a_0 \right) ^2.\tag*{\ding{177}}
\]
则将\ding{176}\ding{177}代入\ding{175}式,得
\begin{align*}
\left( b-a_0 \right) ^2 &\leqslant \left( a-a_0 \right) ^2-2\lambda ^2\left( a-a_0 \right) ^2+\lambda ^2\left( a-a_0 \right) ^2
\\
&=\left( 1-\lambda ^2 \right) \left( a-a_0 \right) ^2.
\end{align*}

(III)由\ding{174}式,可知
\begin{align*}
\left[ f\left( b \right) \right] ^2 &=\left[ f\left( b \right) -f\left( a \right) +f\left( a \right) \right] ^2
\\
&=\left[ f\left( b \right) -f\left( a \right) \right] ^2+2f\left( a \right) \left[ f\left( b \right) -f\left( a \right) \right] +\left[ f\left( a \right) \right] ^2
\\
&\leqslant \left( b-a \right) ^2-2\cdot \frac{b-a}{\lambda}\left[ f\left( b \right) -f\left( a \right) \right] +\left[ f\left( a \right) \right] ^2\quad \text{(用\ding{173}式)}
\\
&=\lambda ^2\left[ f\left( a \right) \right] ^2-\frac{2}{\lambda}\left( b-a \right) \left[ f\left( b \right) -f\left( a \right) \right] +\left[ f\left( a \right) \right] ^2
\\
&\leqslant \lambda ^2\left[ f\left( a \right) \right] ^2-\frac{2}{\lambda}\cdot \lambda \cdot \left( b-a \right) ^2+\left[ f\left( a \right) \right] ^2\quad \text{(用\ding{172}式)}
\\
&=\lambda ^2\left[ f\left( a \right) \right] ^2-2\lambda ^2\left[ f\left( a \right) \right] ^2+\left[ f\left( a \right) \right] ^2=\left( 1-\lambda ^2 \right) \left[ f\left( a \right) \right] ^2.
\end{align*}
\end{proof}

\begin{example}
(2015年江苏)设$a_1,a_2,a_3,a_4$是各项为正数且公差为$d\, (d\neq 0)$的等差数列.
\begin{enumerate}
\item[(1)] 证明: $2^{a_1},2^{a_2},2^{a_3},2^{a_4}$依次构成等比数列;

\item[(2)] 是否存在$a_1,d$,使得$a_1,a_2^2,a_3^3,a_4^4$依次构成等比数列,并说明理由;

\item[(3)] 是否存在$a_1,d$及正整数$n,k$,使得$a_1^n,a_2^{n+k},a_3^ {n+2k},a_4^{n+3k}$依次构成等比数列?并说明理由.
\end{enumerate}
\end{example}
\begin{proof}
(1)证明:因为$\frac{2^{a_{n+1}}}{2^{a_n}}=2^{a_{n+1}-a_n}=2^d\,(n=1,2,3)$是同一个常数,所以$2^{a_1},2^{a_2},2^{a_3},2^{a_4}$依次构成等比数列.

(2)令$a_1+d=a$,则$a_1,a_2,a_3,a_4$分别为$a-d,a,a+d,a+2d\,(a>d,a>-2d,d\neq 0)$.

假设存在$a_1,d$,使得$a_1,a_2^2,a_3^3,a_4^4$依次构成等比数列,则$a^4=(a-d)(a+d)^3$,且$(a+d)^6=a^2(a+2d)^4$,

令$t=\frac{d}{a}$,则$1=(1-t)(1+t)^3$,且$(1+t)^6=(1+2t)^4\, \left( -\frac{1}{2}<t<1,t\ne 0 \right)$,化简得$t^3+2t^2-2=0$ ($\ast$),且$t^2=t+1$.将$t^2=t+1$代入($\ast$)式, $t(t+1)+2(t+1)-2=t^2+3t=t+1+3t=4t+1=0$,
则$t=-\frac{1}{4}$.

显然$t=-\frac{1}{4}$不是上面方程的解,矛盾,所以假设不成立.

因此不存在$a_1,d$,使得$a_1,a_2^2,a_3^3,a_4^4$依次构成等比数列.

(3)假设存在$a_1,d$及正整数$n,k$,使得$a_1^n,a_2^{n+k},a_3^ {n+2k},a_4^{n+3k}$依次构成等比数列,则$a_1^n(a_1+2d)^{n+2k}=(a_1+d)^{2(n+k)}$,
且$(a_1+d)^{n+k}(a_1+3d)^{n+3k}=(a_1+2d)^{2(n+2k)}$.

分别在两个等式的两边同除以$a_1^{2(n+k)}$及$a_1^{2(n+2k)}$,并令$t=\frac{d}{a_1}\, \left(t>-\frac{1}{3},t\ne 0 \right)$,

则$(1+2t)^{n+2k}=(1+t)^{2(n+k)}$,
且$(1+t)^{n+k}(1+3t)^{n+3k}=(1+2t)^{2(n+2k)}$.

将上述两个等式两边取对数,得$(n+2k)\ln (1+2t)=2(n+k)\ln (1+t)$,
且$(n+k)\ln (1+t)+ (n+3k)\ln (1+3t)=2(n+2k)\ln (1+2t)$.

化简得
$$
2k\left[ \ln \left( 1+2t \right) -\ln \left( 1+t \right) \right] =n\left[ 2\ln \left( 1+t \right) -\ln \left( 1+2t \right) \right]$$

$$
3k\left[ \ln \left( 1+3t \right) -\ln \left( 1+t \right) \right] =n\left[ 3\ln \left( 1+t \right) -\ln \left( 1+3t \right) \right],
$$
再将这两式相除,化简得
\[
\ln \left( 1+3t \right) \ln \left( 1+2t \right) +3\ln \left( 1+2t \right) \ln \left( 1+t \right) =4\ln \left( 1+3t \right) \ln \left( 1+t \right),
\tag*{($\ast\ast$)}
\]

$$
g\left( t \right) =4\ln \left( 1+3t \right) \ln \left( 1+t \right) -\ln \left( 1+3t \right) \ln \left( 1+2t \right) -3\ln \left( 1+2t \right) \ln \left( 1+t \right),
$$

$$
g'\left( t \right) =\frac{2\left[ \left( 1+3t \right) ^2\ln \left( 1+3t \right) -3\left( 1+2t \right) ^2\ln \left( 1+2t \right) +3\left( 1+t \right) ^2\ln \left( 1+t \right) \right]}{\left( 1+t \right) \left( 1+2t \right) \left( 1+3t \right)}.
$$

$$
\varphi \left( t \right) =\left( 1+3t \right) ^2\ln \left( 1+3t \right) -3\left( 1+2t \right) ^2\ln \left( 1+2t \right) +3\left( 1+t \right) ^2\ln \left( 1+t \right),
$$

$$
\varphi' \left( t \right) =6\left[ \left( 1+3t \right) \ln \left( 1+3t \right) -2\left( 1+2t \right) \ln \left( 1+2t \right) +\left( 1+t \right) \ln \left( 1+t \right) \right].
$$
令$\varphi_1 \left( t \right)=\varphi' \left( t \right)$,则$\varphi' _1\left( t \right) =6\left[ 3\ln \left( 1+3t \right) -4\ln \left( 1+2t \right) +\ln \left( 1+t \right) \right]$.

令$\varphi_2 \left( t \right)=\varphi'_1 \left( t \right)$,则
$$
\varphi' _2\left( t \right) =\frac{12}{\left( 1+t \right) \left( 1+2t \right) \left( 1+3t \right)}>0.
$$
由$g(0)=\varphi(0)=\varphi_1(0)=\varphi_2(0)=0,\varphi' _2\left( t \right)>0$,知$\varphi_2(t),\varphi_1(t),\varphi(t),g(t)$
在$\left( -\frac{1}{3},0 \right)$和$(0,+\infty)$上均单调.

故$g(t)$只有唯一零点$t=0$,即方程$(\ast\ast)$只有唯一解$t=0$,故假设不成立.

所以不存在$a_1,d$及正整数$n,k$,使得$a_1^n,a_2^{n+k},a_3^ {n+2k},a_4^{n+3k}$依次构成等比数列.
\end{proof}

\begin{example}
(2010年广东)设$A(x_1,y_1),B(x_2,y_2)$是平面直角坐标系$xOy$上的两点,现定义由点$A$到点$B$的一种折线距离$\rho (A,B)$为$\rho (A,B)=|x_2-x_1|+|y_2-y_1|$.对于平面$xOy$上给定的不同的两点$A(x_1,y_1),B(x_2,y_2)$.
\begin{enumerate}
\item[(I)] 若点$C(x,y)$是平面$xOy$上的点,试证明: $\rho (A,C)+\rho(C,B)\geqslant \rho(A,B)$;

\item[(II)] 在平面$xOy$上是否存在点$C(x,y)$同时满足

\ding{172} $\rho (A,C)+\rho(C,B)\geq \rho(A,B)$; \qquad \ding{173} $\rho (A,C)=\rho(C,B)$.

若存在,请求出所有符合条件的点;若不存在,请予以证明.
\end{enumerate}
\end{example}
\begin{proof}
(I)证明:因为$\rho (A,C)=|x-x_1|+|y-y_1|$, $\rho (C,B)=|x_2-x|+|y_2-y|$, $\rho (A,B)=|x_2-x_1|+|y_2-y_1|$,所以
\begin{align*}
\rho \left( A,C \right) +\rho \left( C,B \right) &=\left| x-x_1 \right|+\left| y-y_1 \right|+\left| x_2-x \right|+\left| y_2-y \right|
\\
&=\left( \left| x-x_1 \right|+\left| x_2-x \right| \right) +\left( \left| y-y_1 \right|+\left| y_2-y \right| \right)
\\
&\geqslant \left| \left( x-x_1 \right) +\left( x_2-x \right) \right|+\left| \left( y-y_1 \right) +\left( y_2-y \right) \right|
\\
&=\left| x_2-x_1 \right|+\left| y_2-y_1 \right|=\rho \left( A,B \right).
\end{align*}
(II)解:注意到点$A(x_1,y_1)$与点$B(x_2,y_2)$不同,下面分三种情形讨论.

(i)若$x_1=x_2$,则$y_1\neq y_2$,由条件\ding{173}得
$$
\left| x-x_1 \right|+\left| y-y_1 \right|=\left| x_2-x \right|+\left| y_2-y \right|,
$$
即$\left| y-y_1 \right|=\left| y_2-y \right|$,所以$y=\frac{y_1+y_2}{2}$.

由条件\ding{172}得
$$
\left| x-x_1 \right|+\left| y-y_1 \right|+\left| x_2-x \right|+\left| y_2-y \right|=\left| x_2-x_1 \right|+\left| y_2-y_1 \right|.
$$
所以
$$
2\left| x-x_1 \right|+\frac{1}{2}\left| y_2-y_1 \right|+\frac{1}{2}\left| y_2-y_1 \right|=\left| y_2-y_1 \right|,
$$
所以$\left| x-x_1 \right|=0$,所以$x=x_1$.

因此,所求的点$C$为$\left( x_1,\frac{y_1+y_2}{2}\right)$.

(ii)若$y_1=y_2$,则$x_1\neq x_2$,类似于(i),可得符合条件的点$C$为$\left(\frac{x_1+x_2}{2},y_1\right)$.

(iii)当$x_1\neq x_2$且$y_1\neq y_2$时,不妨设$x_1<x_2$.

(1)若$y_1<y_2$,则由(I)中的证明知,要使条件\ding{172}成立,当且仅当$(x-x_1)(x_2-x)\geqslant 0$与$(y-y_1)(y_2-y)\geqslant 0$同时成立,故$x_1\leqslant x\leqslant x_2$且$y_1\leqslant y\leqslant y_2$.

从而由条件\ding{173},得$x+y=\frac{1}{2}(x_1+x_2+y_1+y_2)$.

此时所求点$C$的全体为
$$
M=\left\{ \left( x,y \right) \left| x+y=\frac{1}{2}\left( x_1+x_2+y_1+y_2 \right) ,x_1\leqslant x\leqslant x_2\,\mathrm{且}\, y_1\leqslant y\leqslant y_2 \right. \right\}.
$$


(2)若$y_1>y_2$,类似地由条件\ding{172}可得$x_1\leqslant x\leqslant x_2$且$y_2\leqslant y\leqslant y_1$.

从而由条件\ding{173}得$x-y=\frac{1}{2}(x_1+x_2-y_1-y_2)$.

此时所求点的全体为
$$
N=\left\{ \left( x,y \right) \left| x-y=\frac{1}{2}\left( x_1+x_2-y_1-y_2 \right) ,x_1\leqslant x\leqslant x_2\,\mathrm{且}\, y_2\leqslant y\leqslant y_1 \right. \right\}.
$$
\end{proof}


\begin{example}
(2014年辽宁)已知函数$f(x)=(\cos x-x)(\pi+2x)-\frac{8}{3}(\sin x+1)$, $g(x)=3(x-\pi)\cos x-4(1+\sin x)\ln\left(3-\frac{2x}{\pi}\right)$.
证明:
\begin{enumerate}
\item[(I)] 存在唯一$x_0\in \left(0,\frac{\pi}{2}\right)$,使$f(x_0)=0$;

\item[(II)] 存在唯一$x_1\in \left(\frac{\pi}{2},\pi\right)$,使$g(x_1)=0$,且对(1)中的$x_0$有$x_0+x_1<\pi$.
\end{enumerate}
\end{example}
\begin{proof}
(I)当$x\in \left(0,\frac{\pi}{2}\right)$时,
$$
f'\left( x \right) =-\left( 1+\sin x \right) \left( \pi +2x \right) -2x-\frac{2}{3}\cos x,
$$
函数$f(x)$在$\left(0,\frac{\pi}{2}\right)$上为减函数,又$f\left( 0 \right) =\pi -\frac{8}{3}>0,f\left( \frac{\pi}{2} \right) =-\pi ^2-\frac{16}{3}<0$,所以存在唯一$x_0\in \left(0,\frac{\pi}{2}\right)$,使$f(x_0)=0$.

(II)考虑函数
$$
h\left( x \right) =\frac{3\left( x-\pi \right) \cos x}{1+\sin x}-4\ln \left( 3-\frac{2}{\pi}x \right) ,\quad x\in \left[ \frac{\pi}{2},\pi \right],
$$
令$t=\pi-x$,则$x\in \left[ \frac{\pi}{2},\pi \right]$时, $t\in \left[0,\frac{\pi}{2}\right]$.


$$
u\left( t \right) =h\left( \pi -t \right) =\frac{3t\cos t}{1+\sin t}-4\ln \left( 1+\frac{2}{\pi}t \right),
$$

$$
u'\left( t \right) =\frac{3f\left( t \right)}{\left( \pi +2t \right) \left( 1+\sin t \right)},
$$
由(I)得,当$t\in (0,x_0)$时, $u'(t)>0$,当$t\in\left(x_0,\frac{\pi}{2}\right)$时, $u'(t)<0$.

在$(0,x_0)$上$u(t)$是增函数,又$u(0)=0$,从而当$t\in (0,x_0]$时, $u(t)>0$,所以$u(t)$在$(0,x_0]$上无零点.

在$\left(x_0,\frac{\pi}{2}\right)$上$u(t)$是减函数,由$u(x_0)>0.u\left(\frac{\pi}{2}\right)=-4\ln 2<0$,存在唯一的$t_1\in \left(x_0,\frac{\pi} {2}\right)$,使$u(t_1)=0$.

所以存在唯一的$t_1\in \left(x_0,\frac{\pi} {2}\right)$使$u(t_1)=0$.

因此存在唯一的$x_1=\pi-t_1\in \left(\frac{\pi} {2},\pi\right)$,使$h(x_1)=h(\pi -t_1)=u(t_1)=0$.

因为当$x\in \left(\frac{\pi} {2},\pi\right)$时, $1+\sin x>0$,故$g(x)=(1+\sin x)h(x)$与$h(x)$有相同的零点,所以存在唯一的$x_1\in \left(\frac{\pi} {2},\pi\right)$,使$g(x_1)=0$.

因$x_1=\pi-t_1,t_1>x_0$,所以$x_0+x_1<\pi$.
\end{proof}

 

 


\begin{enumerate}
\item 08年江西高考

\item 08年北京高考

\item (2017年天津)设$a\in \mathbb{Z}$,已知定义在$\mathbb{R}$上的函数$f(x)=2x^4+3x^3-3x^2-6x+a$在区间$(1,2)$内有一个零点$x_0$, $g(x)$为$f(x)$的导函数.
\begin{enumerate}
\item 求$g(x)$的单调区间;

\item 设$m\in [1,x_0)\cup (x_0,2]$,函数$h(x)=g(x)(m-x_0)-f(m)$,求证: $h(m)h(x_0)<0$;

\item 求证:存在大于$0$的常数$A$,使得对于任意的正整数$p,q$,且$\frac{p}{q}\in [1,x_0) \cup (x_0,2]$,满足$\left|\frac{p}{q}-x_0\right|\geq \frac{1}{Aq^4}$.
\end{enumerate}


\item 1999高考轧辊

\item 2003高考立体几何题

\item 08年广东高考

\item (2003高考江苏卷压轴题)设$a>0$,如图,已知直线$l:y=ax$及曲线$C:y=x^2$, $C$上的点$Q_1$的横坐标为$a_1\,(0<a_1<a)$.从$C$上的点$Q_n\,(n\geq 1)$作直线平行于$x$轴,交直线$l$于点$P_{n+1}$,再从点$P_{n+1}$作直线平行于$y$轴,交曲线$C$于点$Q_{n+1}$. $Q_n\,(n=1,2,3,\cdots)$的横坐标构成数列$\{a_n\}$.
\begin{enumerate}
\item 试求$a_{n+1}$与$a_n$的关系,并求$\{a_n\}$的通项公式;
\item 当$a=1,a_1\leq \frac{1}{2}$时,证明$\sum_{k=1}^{n}(a_k-a_{k+1})a_{k+2}<\frac{1}{32}$;

\item 当$a=1$时,证明$\sum_ {k=1}^{n}(a_k-a_{k+1})a_{k+2}<\frac{1}{3}$.
\end{enumerate}


\item (2010年全国2导数)设函数$f(x)=1-e^{-x}$.
\begin{enumerate}
\item 证明:当$x>-1$时, $f(x)\geq \frac{x}{x+1}$;

\item 设当$x\geq 0$时, $f(x)\leq\frac{x}{ax+1}$,求$a$的取值范围.
\end{enumerate}

\item (2014年全国2导数)已知函数$f(x)=e^x-e^{-x}-2x$.
\begin{enumerate}
\item 讨论$f(x)$的单调性;

\item 设$g(x)=f(2x)-4bf(x)$,当$x>0$时, $g(x)>0$,求$b$的最大值;

\item 已知$1.4142<\sqrt{2}<1.4143$,估计$\ln 2$的近似值(精确到$0.001$).
\end{enumerate}

\item (2013年安徽理科数学)设函数$f_n(x)=-1+x+\frac{x^2}{2^2}+\frac{x^3}{3^2}+\cdots+\frac{x^n}{n^2}\,(x\in \mathbb{R},n\in \mathbb{N}_+)$,证明:
\begin{enumerate}
\item 对每个$n\in \mathbb{N}_+$,存在唯一的$x_n\in\left[\frac{2}{3},1\right]$,满足$f_n(x_n)=0$;

\item 对于任意$p\in \mathbb{N}_+$,由(1)中$x_n$构成数列$\{x_n\}$满足$0<x_n-x_{n+p}<\frac{1}{n}$.
\end{enumerate}

\item (2014年安徽理科数学)设实数$c>0$,整数$p>1,n\in\mathbb{N}^+$.
\begin{enumerate}
\item 证明:当$x>1,x\neq 0$时, $(1+x)^p>1+px$;

\item 数列$\{a_n\}$满足$a_1>c^{\frac{1}{p}},a_{n+1}=\frac{p-1}{p}a_n+\frac{c}{p}a_n^{1-p}$,证明: $a_n>a_{n+1}>c^ {\frac{1}{p}}$.
\end{enumerate}

\item (2013年安徽理科数学)某高校数学系计划在周六和周日各举行一次主主题不同的心理测试活动.分别由李老师和张老师负责,已知该系共有$n$位学生,每次活动均需该系$k$位学生参加($n$和$k$都是固定的正整数),假设李老师和张老师分别将各自活动通知的信息独立、随机地发给该系$k$位学生,且所发信息都能收到,记该系收到李老师或张老师所发活动信息的学生人数为$X$.
\begin{enumerate}
\item 求该系学生甲收到李老师或张老师所发活动通知信息的概率;

\item 求使$P(X=m)$取得最大值的整数$m$.
\end{enumerate}

\item (2010年江西)


\item (2010年江苏)已知$\triangle ABC$的三边长为有理数.
证明:
\begin{enumerate}
\item $\cos A$为有理数;
\item $\cos nA$为有理数.
\end{enumerate}


\item (2011年浙江)设函数$f(x)=(x-a)^2\ln x,a\in \mathbb{R}$. 
\begin{enumerate}
\item 若$x=e$为$y=f(x)$的极值点,求实数$a$;

\item 求实数$a$的取值范围,使得对任意的$x\in (0,3a]$,恒有$f(x)\leq 4e^2$成立. 注: $e$为自然对数的底数.
\end{enumerate}


\item (2018年浙江)


\item (2009年江西)


\item (2004年江苏)已知函数$f(x)\, (x\in\mathbb{R})$满足下列条件:对任意的实数$x_1,x_2$都有
\[\lambda (x_1-x_2)^2\leq (x_1-x_2)[f(x_1)-f(x_2)]\]

\[|f(x_1)-f(x_2)|\leq |x_1-x_2|,\]
其中$\lambda$是大于$0$的常数.设实数$a_0,a,b$满足$f(a_0)=0$和$b=a-\lambda f(a)$.
\begin{enumerate}
\item 证明$\lambda\leq 1$,并且不存在$b_0\neq a_0$,使得$f(b_0)=0$;

\item 证明$(b-a_0)^2\leq (1-\lambda^2)(a-a_0)^2$;

\item 证明: $[f(b)]^2\leq (1-\lambda^2)[f(a)]^2$.
\end{enumerate}

\item (2006年江苏)设数列$\{a_n\},\{b_n\},\{c_n\}$满足: $b_n=a_n-a_{n+2},c_n=a_n+2a_{n+1}+3a_{n+2}\,(n=1,2,3,\cdots)$.

证明: $\{a_n\}$为等差数列的充分必要条件是$\{c_n\}$为等差数列,且 $b_n\leq b_{n+1}\,(n=1,2,3,\cdots)$. (听闻此题当年全江苏省只有几十位考生拿到一半以上的分数,只有不到10人拿满分)

 

\item (2011年江苏)设$M$为部分正整数组成的集合,数列$\{a_n\}$的首项$a_1=1$,前$n$项的和为$S_n$.已知对任意的整数$k\in M$,当整数$n>k$时, $S_{n+k}+S_{n-k}=2(S_n+S_k)$都成立.
\begin{enumerate}
\item 设$M=\{1\},a_2=2$,求$a_5$的值;

\item 设$M=\{3,4\}$,求数列$\{a_n\}$的通项公式.
\end{enumerate}

这个题目的结论可以推广到:

数列$\{a_n\}$对互素的$k_1,k_2\in \mathbb{N}^\ast,k_1>k_2>0$满足:
\[a_{n+k_1}+a_{n-k_1}=2a_n\,(n>k_1),\quad a_{n+k_2}+a_{n-k_2}=2a_n\,(n>k_2),\]
则此时数列$\{a_n\}$是等差数列.

\item (2015年江苏)设$a_1,a_2,a_3,a_4$是各项为正数且公差为$d\, (d\neq 0)$的等差数列.
\begin{enumerate}
\item 证明: $2^{a_1},2^{a_2},2^{a_3},2^{a_4}$依次成等比数列;

\item 是否存在$a_1,d$,使得$a_1,a_2^2,a_3^3,a_4^4$依次成等比数列,并说明理由;

\item 是否存在$a_1,d$以及正整数$n,k$,使得$a_1^n,a_2^{n+k},a_3^ {n+2k},a_4^{n+3k}$依次成等比数列,并说明理由.
\end{enumerate}

\item (2012年安徽)数列$\{x_n\}$满足$x_1=0,x_{n+1}=-x_n+x_n+c\, (n\in \mathbb{N}^\ast)$.
\begin{enumerate}
\item 证明: $\{x_n\}$是递减数列的充分必要条件是$c<0$;
\item 求$c$的取值范围,使$\{x_n\}$是递增数列.
\end{enumerate}

\item (2010年广东)设$A(x_1,y_1),B(x_2,y_2)$是平面直角坐标系$xOy$上的两点,现定义由点$A$到点$B$的一种折线距离$\rho (A,B)$为$p(A,B)=|x_2-x_1|+|y_2-y_1|$.对于平面$xOy$上给定的不同的两点$A(x_1,y_1),B(x_2,y_2)$.
\begin{enumerate}
\item 若点$C(x,y)$是平面$xOy$上的点,试证明$\rho (A,C)+\rho(C,B)\geq \rho(A,B)$;

\item 在平面$xOy$上是否存在点$C(x,y)$同时满足

\ding{172} $\rho (A,C)+\rho(C,B)\geq \rho(A,B)$; \qquad \ding{173} $\rho (A,C)=\rho(C,B)$.

若存在,请求出所有符合条件的点,请予以证明.
\end{enumerate}

\item (2009年湖南)


\item (2015年广东)调和数列,类似2014年陕西.

\item (2014年辽宁)已知函数$f(x)=(\cos x-x)(\pi+2x)-\frac{8}{3}(\sin x+1)$, $g(x)=3(x-\pi)\cos x-4(1+\sin x)\ln\left(3-\frac{2x}{\pi}\right)$.
证明:
\begin{enumerate}
\item 存在唯一$x_0\in \left(0,\frac{\pi}{2}\right)$,使$f(x_0)=0$;

\item 存在唯一$x_1\in \left(\frac{\pi}{2},\pi\right)$,使$g(x_1)=0$,且对(1)中的$x_0$有$x_0+x_1<\pi$.
\end{enumerate}

\item (2019年北京高考)已知函数 $f(x)=\frac{1}{4} x^{3}-x^{2}+x$.
\begin{enumerate}%[itemsep=-0.3em,topsep=0pt,labelsep=.5em,leftmargin=1.7em]
\item 求曲线$y=f(x)$的斜率为1的切线方程;

\item 当$x\in [-2,4]$时,求证: $x-6\leqslant f(x)\leqslant x$;

\item 设$F(x)=|f(x)-(x+a)|\, (a \in \mathbb{R})$,记$F(x)$在区间$[-2,4]$上的最大值为$M(a)$.当$M(a)$最小时,求$a$的值.
\end{enumerate}
\begin{solution}
\textbf{解法一.} (III)由(II)知,当$a<-3$时, $M(a)\geq F(0)=|g(0)-a|=-a>3$;

当$a>-3$时, $M(a)\geq F(-2)=|g(-2)-a|=6+a>3$;

当$a=-3$时, $M(a)=3$.

综上,当$M(a)$最小时, $a=-3$.

\textbf{解法二.}由$F(-2)=|a+6|,F(4)=|a|$,则
\[
M\left( a \right) \ge \frac{F\left( -2 \right) +F\left( 4 \right)}{2}\ge \frac{\left| a+6+\left( -a \right) \right|}{2}=3,
\]
当$a=-3$时取等号成立.
\end{solution}


\textcolor{red}{切比雪夫最佳逼近线:}求形如函数$F(x)=|f(x)-ax-b|$的最大值的最小值问题.

定义集合$A=\{g(x)=ax+b|a,b\in \mathbb{R}\}$,若存在函数 $g_0(x)\in A$使得对任意$g(x)\in A$,都满足
\[\max_{m\leq x\leq n}|f(x)-g_0(x)|\leq \min_{a,b\in \mathbb{R}}\max_{m\leq x\leq n}|f(x)-g(x)|,\]
则称$g_0(x)$为$f(x)$的最佳逼近直线.


\item 设函数$f(x)=|x^3-6x^2+ax+b|$,若对任意的实数$a$和$b$,总存在$x_0\in [0,3]$,使得$f(x_0)\geq m$,求实数$m$的最⼤值. 来源:2020 年 1 ⽉清华⼤学 THUSSAT 测试(理)

考虑问题的反⾯,我们选择合适的$a,b$,使得函数$f(x)$在$[0,3]$上的最⼤值尽可能的小,这个最大值的最小值即所求.

令$g(x)=x^3-6x^2$,则$g'(x)=3x^2-12x$,而经过两端点$(0,g(0))$和$(3,g(3))$的直线斜率为$\frac{g(3)-g(0)}{3-0}=-9$.令$g'(x)=3x^2-12x=-9$,解得$x=1$.

又$f(0)=|b|,f(3)=|-27+3a+b|,f(1)=|-5+a+b|$,于是
\[\frac{f(3)+3f(1)+2f(0)}{6}\geq \frac{|(-27+3a+b)-3(-5+a+b)+2b|}{6}=2.\]
这就证明了$f(x)$的最⼤值的最⼩值不⼩于$2$.


\item (2019年北京高考)已知抛物线$C:x^2=-2py$经过点$(2,-1)$.
\begin{enumerate}%[itemsep=-0.3em,topsep=0pt,labelsep=.5em,leftmargin=1.7em]
\item 求抛物线$C$的方程及其准线方程;
\item 设$O$为原点,过抛物线$C$的焦点作斜率不为$0$的直线$l$交抛物线$C$于两点$M,N$,直线$y=-1$分别交直线$OM,ON$于点$A$和点$B$.求证:以$AB$为直径的圆经过$y$轴上的两个定点.
\end{enumerate}


以$AB$为\textcolor{red}{直径}的圆$\to$两直线\textcolor{red}{垂直}$\to$ 向量$\overrightarrow{DA}\cdot \overrightarrow{DB}=0$或斜率之积等于$-1$.

\item (2017年北京高考)已知函数$f(x)=e^x\cos x-x$.
\begin{enumerate}
\item[(I)] 求曲线$y=f(x)$在点$(0,f(0))$处的切线方程;
\item[(II)] 求函数$f(x)$在区间$\left[0,\frac{\pi}{2} \right]$上的最大值和最小值.
\end{enumerate}

令导函数$f'(x)=e^x(\cos x-\sin x)-1=0$,得到无法求解的超越方程.

分析极值不一定要解方程$f'(x)=0$,\textcolor{red}{一阶导不够,二阶导来凑:}\,利用二阶导分析(一阶)导函数的单调性.

\item 已知函数$f(x)=e^x-ax^2\,(a\in \mathbb{R})$.
\begin{enumerate}
\item[(I)] 若曲线$y=f(x)$在$(1,f(1))$处的切线与$x$轴平行,求$a$;

\item[(II)] 已知$f(x)$在$[0,1]$上的最大值不小于$2$,求$a$的取值范围;

\item[(III)] 写出$f(x)$所有可能的零点个数及相应的$a$的取值范围.
\end{enumerate}

\item 设函数$f(x)=e^x-1-x-ax^2$.

(1)若$a=0$,求$f(x)$的单调区间;

(2)若当$x\geq 0$时$f(x)\geq 0$,求$a$的取值范围.

函数不等式: $e^x\geq x+1,\ln (x+1)\leq x$与$\sin x \leq x\leq \tan x$.

泰勒公式:
\[e^x=1+x+\frac{x^2}{2!}+\frac{x^3}{3!}+\cdots+\frac{x^n}{n!}+\cdots\]

\item 已知函数$f(x)=ax+\ln x+1$.

(1)讨论函数$f(x)$零点个数;

(2)对任意的$x>0,f(x)\leq xe^{2x}$恒成立,求实数$a$的取值范围.

设而不求思想:隐零点$x_0$满足$2x_0^2e^{2x_0}+\ln x_0=0$,则$0<x_0<1$.

\item (2020年西城一模)对于正整数$n$,如果$k\,(k\in \mathbb{N}^\ast)$个整数$a_1,a_2,\cdots,a_k$满足$1\leq a_1\leq a_2\leq \cdots\leq a_k\leq n$,且$a_1+a_2+\cdots+a_k=n$,则称数组$(a_1,a_2,\cdots,a_k)$为$n$的一个“正整数分拆”.记$a_1,a_2,\cdots,a_k$均为偶数的“正整数分拆”的个数为$f_n$, $a_1,a_2,\cdots,a_k$均为奇数的“正整数分拆”的个数为$g_n$.

(I)写出整数4的所有“正整数分拆”;

(II)对于给定的整数$n\,(n\geq 4)$,设$(a_1,a_2,\cdots,a_k)$是$n$的一个“正整数分拆”
且$a_1=2$,求$k$的最大值;

(III)对所有的正整数$n$,证明: $f_n<g_n$;并求出使得等号成立的$n$的值.

(注:对于$n$的两个“正整数分拆”$(a_1,a_2,\cdots,a_k)$与$(b_1,b_2,\cdots,b_m)$,当且仅当$k=m$且$a_1=b_1,a_2=b_2,\cdots,a_k=b_m$时,称这两个“正整数分拆”是相同的.)

\item (2012年海淀二模)将一个正整数$n$表示为$a_1+a_2+\cdots+a_p\,(p\in \mathbb{N}^\ast)$的形式,其中$a_i\in \mathbb{N}^\ast,i=1,2,\cdots,p$,且$a_1\leq a_2\leq \cdots\leq a_p$,记所有这样的表示法的种数为$f(n)$(如$4=4,4=1+3,4=2+2,4=1+1+2,4=1+1+1+1$,故$f(4)=5$).

(1)写出$f(3),f(5)$的值,并说明理由;

(2)对任意正整数$n$,比较$f(n+1)$与$\frac{1}{2}[f(n)+f(n+2)]$的大小,并给出证明;

(3)当正整数$n>6$时,求证: $f(n)\geq 4n-13$.


\item (2020年西城一模)设函数
\[\begin{cases}
x^2+10x+1, & x\leq 0 \\
|\lg x|, & x>0
\end{cases}\]
若关于$x$的方程$f(x)= a\,(a\in \mathbb{R})$有四个实数解$x_i\,(i=1,2,3,4)$,其中$x_1<x_2<x_3<x_4$.则$(x_1+x_2)(x_3-x_4)$的取值范围是

(A) $(0,101]$\qquad (B) $(0,99]$ \qquad (C) $(0,100]$ \qquad (D) $(0,+\infty)$

\item

\item

\item

\item

\item

\item

\item

\item
\end{enumerate}

设$f(x)=e^{mx-1}-\frac{\ln x}{x}$,若$f(x)$的最小值为$m$,求$m$的最小值.

%https://wenku.baidu.com/view/09e8b0215901020207409c2a.html?rec_flag=default&sxts=1566951788471


%https://wenku.baidu.com/view/1bd3fb6648d7c1c708a145b2.html?sxts=1566951843376

%https://wenku.baidu.com/view/aecb43a1b0717fd5360cdc33.html?rec_flag=default&sxts=1566951578646

 

试举几例:

2010年北京高考数学压轴题命题背景是纠错码理论中的Plotkin上界(有所改编);

2014年北京高考数学压轴题命题背景是多工序流水线最优化排序问题中的Johnson法则;

2015年北京高考数学压轴题命题背景有限交换环上迭代图的一个特例;

两篇参考文献:

1.北京高考数学压轴题的教学实践与反思,《数学通报》杂志2017年第1期pp45-pp48(截图见下方);

2.再谈2015年北京高考数学压轴题与“数学黑洞”问题,中学数学杂志, 2017年第1期

3.从高中数学试题到纠错码理论,李启超,荣贺
%作者:饮冰
%链接:https://www.zhihu.com/question/57845091/answer/155345007


\section{导数压轴题}


\subsection{参变分离}


\subsection{导数不等式}
%https://zhuanlan.zhihu.com/p/91032042
%https://zhuanlan.zhihu.com/p/51584482

\begin{theorem}{函数不等式链}{1}
当$x\geq 0$时,
\[
\frac{x}{x+1}\le \frac{2x}{x+2}\le \ln \left( x+1 \right) \le \frac{1}{2}\left( x+1-\frac{1}{x+1} \right) \le x.
\]
\end{theorem}


\subsection{设而不求:隐零点}

\subsection{极值点偏移}
%https://zhuanlan.zhihu.com/p/32842987

\begin{theorem}{指、对数平均不等式}{zdpjz}
当实数$a\neq b$时,有
\[e^{\frac{a+b}{2}}<\frac{e^a-e^b}{a-b}<\frac{e^a+e^b}{2}.\]

\[\sqrt{ab}<\frac{a-b}{\ln a-\ln b}<\frac{a+b}{2}.\]
\end{theorem}

\begin{example}
(2013年陕西)已知函数$f(x)=e^x,x\in \mathbb{R}$.

(1)若直线$y=kx+1$与$f(x)$的反函数的图像相切,求实数$k$的值;

(2)设$x>0$,讨论曲线$y=f(x)$与曲线$y=mx^2\,(m>0)$公共点的个数;

(3)设$a<b$,比较$\frac{f(a)+f(b)}{2}$与$\frac{f(b)-f(a)}{b-a}$的大小,并说明理由.
\end{example}
\begin{solution}

\end{solution}


\begin{example}
(2016新课标1)已知函数$f(x)=(x-2)e^x+a(x-1)^2$有两个零点.

(I)求$a$的取值范围;

(II)设$x_1,x_2$是$f(x)$的两个零点,证明: $x_1+x_2<2$.
\end{example}
\begin{solution}

\end{solution}

\begin{example}
(2018皖南八校第三次联考理科数学)
已知函数$f(x)=e^x-x^2-ax$有两个极值点$x_1,x_2\,(x_1<x_2)$.

(1)求$a$的取值范围;

(2)求证: $e^{x_1}+e^{x_2}>4$.
\end{example}
\begin{solution}
\[\frac{e^{x_1}-e^{x_2}}{x_1-x_2}=2<\frac{e^{x_1}+e^{x_2}}{2}.\]
\end{solution}

\begin{theorem}{Hermite-Hadamard不等式}{hhbds}
若函数$f(x)$在$[a,b]$上的二阶导数非负,则有:
\[
f\left( \frac{a+b}{2} \right) \le \frac{1}{b-a}\int_a^b{f\left( x \right) dx}\le \frac{f\left( a \right) +f\left( b \right)}{2},
\]
当且仅当$f(x)$是一次函数时取等号成立.
\end{theorem}

\begin{example}
(匈牙利, 1914)设$f(x)=ax^2+bx+c$, $a,b,c$为实数,如果对于所有适合$-1\leq x\leq 1$的$x$值,都有$-1\leq f(x)\leq 1$成立,则对这些$x$的值有$-4\leq 2ax+b\leq 4$.
\end{example}

此题的背景是切比雪夫多项式的马尔科夫定理:如果具有实系数的$n$次多项式
\[f(x)=a_0+a_1x+a_2x^2+\cdots+ a_nx^n\]
对所有的$-1\leq x\leq 1$满足不等式
\[-1\leq f(x)\leq 1.\]
那么它的导函数满足不等式
\[-n^2\leq f'(x)\leq n^2.\]

虽然背景是高等的,但解法只用到一次函数$g(x)=2ax+b$的单调性、取值的技巧和不等式的放缩运算.
\begin{solution}
$12$.
\end{solution}
%切比雪夫多项式的马尔科夫定理,https://wenku.baidu.com/view/d0c9e2bbfd0a79563c1e720f.html

%https://zhuanlan.zhihu.com/p/105766114

\subsection{切比雪夫多项式}
%https://zhuanlan.zhihu.com/p/105766114

利用三角函数$n$倍角公式
\begin{align*}
\cos(0)&=1,\\
\cos(x)&=\cos x,\\
\cos(2x)&=2\cos^2 x-1,\\
\cos(3x)&=4\cos^3 x-3\cos x,\\
\cos(4x)&=8\cos^4 x-8\cos^2 x+1,\\
\cos(5x)&=16\cos^5 x-20\cos^3 x+5\cos x,\\
\end{align*}
可知$\cos (n\theta)$可以表示成$\cos\theta$的多项式, $T_n(x)=\cos(n\cdot \arccos x)$是一个$n$次多项式,称为$n$次切比雪夫多项式,其中$x\in [-1,1],n\in \mathbb{N}$.于是
\begin{align*}
T_0(x) &=1,\\
T_1(x) &=x,\\
T_2(x) &=2x^2-1,\\
T_3(x) &=4x^3-3x,\\
T_4(x) &=8x^4-8x^2+1,\\
T_5(x) &=16x^5-20x^3+5x,\\
\end{align*}

性质1. $T_n(x)$在$[-1,1]$中有$n$个不同的实根$x_k=\cos\frac{(2k-1)\pi}{2n},k=1,2,3,\cdots,n$.

性质2. $T_n(x)$在$[-1,1]$中有$n+1$个点$x_k^\ast=\cos\frac{k\pi}{n},k=0,1,2,3,\cdots,n$,轮流取最大值$1$和最小值$-1$.例如:当$n=2$时, $x_k^\ast=-1,0,1$.当$n=3$时, $x_k^\ast=-1,-\frac{1}{2},\frac{1}{2},1$.

性质3. $T_n(x)$满足递推关系$T_0(x)=1,T_1(x) =x$,
\[T_{n+1}(x)=2xT_n(x)-T_{n-1}(x),\]
其母函数为
\[\sum_{n=0}^{\infty}T_n(x)t^n=\frac{1-tx}{1-2tx+t^2}.\]

定理.对任意$n$次首一多项式$P(x)$,设$M=\max_{x\in[-1,1]}|P(x)|$,则$M_{\min}=\frac{1}{2^{n-1}}$.

证明.引理:设$n$次首一多项式$Q(x)$的$n$个根$\alpha_1,\alpha_2,\cdots,\alpha_n$均属于$(-1,1)$.在$[-1,\alpha_1),(\alpha_1,\alpha_2),\cdots,
(\alpha_{n-1},\alpha_n),(\alpha_n,1]$内各取一点$\beta_0,\beta_1,\cdots,\beta_n$,则对任意首一多项式$R(x)$,均有
\[\max_ {x\in[-1,1]}|R(x)|\geq \min_ {0\leq i\leq n}|Q(\beta_i)|.\]

引理的证明: (反证法)设存在$R(x)$使得
\[\max_ {x\in[-1,1]}|R(x)|< \min_ {0\leq i\leq n}|Q(\beta_i)|\triangleq C.\]
于是$R(x)\in (-C,C),\forall x\in [-1,1]$.

考虑$T(x)=R(x)-Q(x)$,则数列$T(\beta_0),T(\beta_1),\cdots,T(\beta_n)$必定正负交错(如图),则$T_n$有至少$n$个根.

然而$R(x),Q(x)$均为首一多项式,故$T(x)\equiv 0$.则$R(x)=Q(x)$,显然矛盾.

回到原题.设$T_n(x)$为$n$次切比雪夫多项式,令$Q(x)=\frac{1}{2^{n-1}}T_n(x)$,则$Q(x)$的各零点$\alpha_i=\cos\frac{(2i-1)\pi}{n}(i=1,2,\cdots,n)$均属于$(-1,1)$.

在引理中取$\beta_i=\cos\frac{i\pi}{n}(i=0,1,\cdots,n)$,即得$M\geq \frac{1}{2^{n-1}}$,当$P(x)\equiv Q(x)$时可取等.

\begin{theorem}{}{}
设$f(x)$为一个$n$次多项式,首项为$ax^n$,定义域为$D$,值域为$I$,用$|D|$表示$D$的区间长度,则$\frac{|I|}{2}\geq 2^{1-2n}\cdot |a|\cdot |D|^n$.事实上,等号成立时, $\frac{|I|}{2}$也就是$|f(x)|_{\max}$的最小值.等号成立的条件为$f(x)$经过平移及伸缩变换使得定义域为$D$的$T_n$.
\end{theorem}

\begin{solution}
我们用$[a,b]$表示定义域,这样$|D|=b-a$.当$a=-1,b=1$时,我们已证明了多项式$T_n(x)$的范数为$\frac{1}{2^{n-1}}$.为了求出它在任意区间$[a,b]$上的范数,必须采用把区间$a\leq y\leq b$映射到区间$-1\leq x\leq 1$的线性变换$x=\frac{2}{b-a}y-\frac{a+b}{b-a}$.此时我们得到多项式
\[
p\left( y \right) =T_n\left( \frac{2}{b-a}y-\frac{a+b}{b-a} \right) =\left( \frac{2}{b-a}y-\frac{a+b}{b-a} \right) ^n+\cdots
\]
它的最高次项系数非$1$而为$\frac{2^n}{(b-a)^n}$.把$p(y)$用这个数来除,我们得到在区间$[a,b]$上的切比雪夫多项式
\[
\widehat{T}_n\left( y \right) =\frac{\left( b-a \right) ^n}{2^n}T_n\left( \frac{2}{b-a}y-\frac{a+b}{b-a} \right).
\]
它的最高项系数已为$1$了.易见,它的范数等于
\[
\lVert \widehat{T}_n\left( y \right) \rVert =\frac{\left( b-a \right) ^n}{2^n}\lVert T_n\left( y \right) \rVert =\frac{\left( b-a \right). ^n}{2^{2n-1}}
\]
最后乘上首项的系数$a$,我们便得到了
\[\frac{|I|}{2}\geq 2^{1-2n}\cdot |a|\cdot |D|^n.\]
\end{solution}

对于切比雪夫最佳逼近直线,有如下常用结论:
\begin{theorem}{切比雪夫最佳逼近直线理论}{}
若函数$f(x)$在区间$[m,n]$上具有二阶导数,且$f''(x)$在区间$[m,n]$上不变号,则$f(x)$的最佳逼近直线为
\[
g\left( x \right) =k\left( x-\frac{m+c}{2} \right) +\frac{f\left( m \right) +f\left( c \right)}{2},
\]
其中$k=\frac{f\left( m \right) -f\left( n \right)}{m-n}$,实数$c$的值由方程$f'(c)=\frac{f\left( m \right) -f\left( n \right)}{m-n}$解得.
\end{theorem}

\subsection{切比雪夫最佳逼近直线}

%https://zhuanlan.zhihu.com/p/130443282

%https://zhuanlan.zhihu.com/p/105766114


\begin{example}
1
\end{example}

\begin{solution}
1
\end{solution}

\begin{example}
(2017年全国高中数学联赛第9题)设$k,m$为实数,不等式$|x^2-kx-m|\leq 1$对所有$x\in [a,b]$成立.证明: $b-a\leq 2\sqrt{2}$.
\end{example}
\begin{solution}
令$f(x)=x^2-kx-m,x\in [a,b]$,则$f(x)\in [-1,1]$.于是
\begin{align*}
f\left( a \right) &=a^2-ka-m\le 1, \tag*{\ding{172}}\\
f\left( b \right) &=b^2-kb-m\le 1,\tag*{\ding{173}}\\
f\left( \frac{a+b}{2} \right) &=\left( \frac{a+b}{2} \right) ^2-k\left( \frac{a+b}{2} \right) -m\ge -1,\tag*{\ding{174}}
\end{align*}
由\ding{172}$+$\ding{173}$-2\times$\ding{174}可知
\[
\frac{\left( a-b \right) ^2}{2}=f\left( a \right) +f\left( b \right) -2f\left( \frac{a+b}{2} \right) \le 4,
\]
故$b-a\leq 2\sqrt{2}$.
\end{solution}


\begin{example}
(2018年全国高中数学联赛B卷二试)设$a,b$是实数,函数$f(x)=ax+b+\frac{9}{x}$.证明:存在$x_0\in [1,9]$,使得$|f(x_0)|\geq 2$.
\end{example}
\begin{solution}
\textbf{证法1.}只需证明存在$u,v\in [1,9]$,满足$|f(u)-f(v)|\geq 4$,进而由绝对值不等式得
\[|f(u)|+|f(v)|\geq |f(u)-f(v)|\geq 4,\]
故$|f(u)|\geq 2$与$|f(v)|\geq 2$中至少有一个成立.

当$a\in \left( -\infty ,\frac{1}{2} \right] \cup \left[ \frac{3}{2},+\infty \right)$时,有
\[|f(1)-f(9)|=|(a+b+9)-(9a+b+1)|=8|1-a|\geq 4.\]

当$\frac{1}{2}<a<\frac{3}{2}$时,有$\frac{3}{\sqrt{a}}\in [1,9]$.再分两种情况:若$\frac{1}{2}<a<1$,则
\[
\left| f\left( 1 \right) -f\left( \frac{3}{\sqrt{a}} \right) \right|=\left| \left( a+b+9 \right) -\left( 6\sqrt{a}+b \right) \right|=\left( 3-\sqrt{a} \right) ^2\geq 4.
\]
若$1<a<\frac{3}{2}$,则
\[
\left| f\left( 9 \right) -f\left( \frac{3}{\sqrt{a}} \right) \right|=\left| \left( 9a+b+1 \right) -\left( 6\sqrt{a}+b \right) \right|=\left( 3\sqrt{a}-1 \right) ^2\geq 4.
\]
综上可知,存在$u,v\in [1,9]$,满足$|f(u)-f(v)|\geq 4$,从而命题得证.


\textbf{证法2.}用反证法.假设对任意$x\in [1,9]$,均有$|f(x)|<2$,则
\[|f(1)|<2,\qquad |f(3)|<2,\qquad |f(9)|<2.\]
易知
\begin{align*}
f(1)&=a+b+9, \tag*{\ding{172}} \\
f(3)&=3a+b+3, \tag*{\ding{173}} \\
f(9)&=9a+b+1. \tag*{\ding{174}}.
\end{align*}
由\ding{172},\ding{173}得$2a-6=f(2)-f(1)$,又由\ding{173},\ding{174}得$6a-2=f(3)-f(2)$.

由上述两式消去$a$,可知
\[f(3)-4f(2)+3f(1)=(6a-2)-3\cdot (2a-6)=16.\]
但$f(3)-4f(2)+3f(1)<2+4\cdot 2+3\cdot 2=16$,矛盾!从而命题得证.
\end{solution}

\begin{example}
(2015年北京大学自主招生试题第3题)已知$|x^2+px+q|\leq 2$对任意$x\in [1,5]$成立,则不超过$\sqrt{p^2+q^2}$的最大整数是\underline{\hspace{2cm}}.
\end{example}

\begin{solution}
1
\end{solution}

\begin{example}
已知函数$f(x)=\left|x+\frac{1}{x}-ax-b\right|\,(a,b\in \mathbb{R})$,当$x\in \left[\frac{1}{2},2\right]$时,设$f(x)$的最大值为$M(a,b)$,则$M(a,b)$的最小值为\underline{\hspace{2cm}}.
\end{example}
\begin{solution}
设$M=f_{\max}(x)$,则
\[\begin{cases}
M\ge f\left( \frac{1}{2} \right) =\left| \frac{5}{2}-\frac{a}{2}-b \right|=\frac{1}{2}\left| a+2b-5 \right|,
\\
M\ge f\left( 1 \right) =\left| 2-a-b \right|,
\\
M\ge f\left( 2 \right) =\left| \frac{5}{2}-2a-b \right|=\frac{1}{2}\left| 4a+2b-5 \right|,
\end{cases}\]

\begin{align*}
2M &\ge \frac{2}{3}f\left( \frac{1}{2} \right) +f\left( 1 \right) +\frac{1}{3}f\left( 2 \right)
\\
&=\frac{1}{3}\left| a+2b-5 \right|+\left| 2-a-b \right|+\frac{1}{6}\left| 4a+2b-5 \right|
\\
&\ge \left| 2-\frac{5}{3}-\frac{5}{6} \right|=\frac{1}{2},
\end{align*}
于是$M(a,b)$的最小值为$\frac{1}{2}$.
\end{solution}

\begin{example}
(2015年浙江高考)已知函数$f(x)=x^2-+ax+b\,(a,b\in \mathbb{R})$,记$M(a,b)$是$|f(x)|$在区间$[-1,1]$上的最大值.

(1)证明:当$|a|\geq 2$时, $M(a,b)\geq 2$;

(2)当$a,b$满足$M(a,b)\leq 2$,求$|a|+|b|$的最大值.
\end{example}
\begin{solution}
1
\end{solution}

\begin{example}
(2015年1月浙江省学业水平考试第34题)设函数$f(x)=\left|\sqrt{x}-ax-b\right|,a,b\in \mathbb{R}$.

(1)当$a=0,b=1$时,写出函数$f(x)$的单调区间;

(2)当$a=\frac{1}{2}$时,记函数$f(x)$在区间$[0,4]$上的最大值为$g(b)$,当$b$变化时,求$g(b)$的最小值;

(3)若对任意实数$a,b$,总存在实数$x_0\in [0,4]$使得不等式$f(x_0)\geq m$成立,求实数$m$的取值范围.
\end{example}

\begin{solution}
(1)单调递减区间是$(0,1)$,单调递增区间是$(1,+\infty)$;

(2)在区间$[0,4]$上$-b\leq h(x)\leq h(1)=\frac{1}{2}-b$.

当$b\leq \frac{1}{4}$, $f(x)$在区间$[0,4]$上的最大值为$g(b)=\frac{1}{2}-b$;

当$b> \frac{1}{4}$, $f(x)$在区间$[0,4]$上的最大值为$g(b)=b$.

当$b=\frac{1}{4}$时, $g(b)$取得最小值$\frac{1}{4}$.

(3)在$[0,4]$上, $-b\leq u(x)\leq 2-4a-b$.

当$b\leq 1-2a$时, $M(b)=2-4a-b$,当$b>1-2a$时, $M(b)=b$.

从而当$a\leq \frac{1}{4}$, $b=1-2a$时$M(b)$取最小值, $M(b)_{\min}=1-2a\geq \frac{1}{2}$;当$a> \frac{1}{4}$, $u(x)$在$\left[0,\frac{1}{4a^2}\right)$上单调递增,在$\left[\frac{1}{4a^2},4\right)$上单调递减.

在$a\in \left[\frac{1}{4},\frac{1}{2}\right]$, $-b\leq u(x)\leq \frac{1}{4a}-b$,当$b=\frac{1}{8a}$时, $M(b)_{\min}=\frac{1}{8a}\geq \frac{1}{4}$;

在$a\in \left(\frac{1}{2},+\infty\right)$, $2-4a-b\leq u(x)\leq \frac{1}{4a}-b$,当$b=1-2a+\frac{1}{8a}$时, $M(b)_{\min}=2a+\frac{1}{8a}-1>\frac{1}{4}$.

综上所述, $M(b)_{\min}=\frac{1}{4}$.

若对任意实数$a,b$,总存在实数$x_0\in [0,4]$使得不等式$f(x_0)\geq m$成立,等价于$m\leq f(x)_{\max}$恒成立, $m\leq \frac{1}{4}$.
\end{solution}

 

 

\begin{example}
(2016年天津高考)设函数$f(x)=x^3-ax-b,x\in \mathbb{R}$,其中$a$、$b\in \mathbb{R}$.

(1)求$f(x)$的单调区间;

(2)若$f(x)$存在极值点$x_0$,且$f(x_1)=f(x_0)$,其中$x_1\neq x_0$.求证: $x_1+2x_0=0$;

(3)设$a>0$,函数$g(x)= |f(x)|$,求证: $g(x)$在区间$[-1,1]$上的最大值不小于$\frac{1}{4}$.
\end{example}
%https://www.zhihu.com/question/345947963/answer/1046248071
\begin{solution}

\end{solution}


\begin{example}
求所有整数$a,b$,其中$|a|\leq 5,|b|\leq 5$,使得$x^4-3x^2-ax+b=0$恰有两个不同的整数解.
\end{example}
\begin{solution}
当$x\geq 0$时,有$x^4-3x^2=ax-b\leq 5x+5$,则$0\leq x\leq 2.43$;同理可知,当$x< 0$时,有$x^4-3x^2=ax-b\leq -5x+5$,
则$0>x>-2.43$.因此整数$x$只能取$-2,-1,0,1,2$.相应地,有$4+2a+b=0,-2+a+b=0,b=0,-2-a+b=0,4-2a+b=0$.

经检验,有$a=-2,b=0$,此时$x=-2,0,1$,矛盾; $a=0,b=-4$,此时$x=-2,1$,满足题意; $a=2,b=0$,此时$x=2,-1$,满足题意; $a=0,b=2$,此时整数$x=-1,1$也满足.

综上, $(a,b)=(0,-4),(2,0)$或$(0,2)$.
\end{solution}

\begin{example}
2.在一次宴会上,有10对夫妻参加,将所有男士安排在一个有10个座位的圆桌旁,所有女士安排在另外一张也是10个座位的圆桌旁,且每位女士的座位相对位置和她的配偶相同,我们发现新冠病毒在与会者之间传播,传播途径如下: $A$为一名健康与会者,当且仅当其座位两侧及其配偶三人间至少有两人感染的情况下, $A$才会被感染.设宴会开始时的$20$人中有$S$人感染,病毒在与会者中传播,则最后可能使所有与会者都感染上的$S$的最小值为多少?
\end{example}

\begin{example}
求$\frac{1}{4\times 1^4+1}+\frac{2}{4\times 2^4+1}+\frac{3}{4\times 3^4+1}+\cdots+\frac{100}{4\times 100^4+1}$的值.
\end{example}

\begin{example}
(2011年清华保送生)证明:对于任意的正整数$a$、$b$有
\[
\left( a,b \right) =\frac{1}{a}\sum_{m=0}^{a-1}{\sum_{n=0}^{a-1}{e^{\frac{2\pi imnb}{a}}}}.
\]
\end{example}
\begin{solution}
设$\mathrm{gcd}(a,b)=d,a=dx,b=dy,w=e^{2\pi i\frac{y}{x}}$,其中$a,b,d,x,y\in \mathbb{Z}_+$,则$w^x=1$.
\[
\frac{1}{a}\sum_{m=0}^{a-1}{\sum_{n=0}^{a-1}{e^{\frac{2\pi imnb}{a}}}}=\frac{1}{a}\sum_{m=0}^{a-1}{\sum_{n=0}^{a-1}{w^{mn}}}.
\]
注意到$\left( 1-w^m \right) \sum_{n=0}^{a-1}{w^{mn}}=1-w^{am}=0$.因此,当且仅当$m=0,x,2x,\cdots,(d-1)x$时, $w^m=1$,此时$\sum_{n=0}^{a-1}{w^{mn}}=a$.而当$m$取其他值时, $\sum_{n=0}^{a-1}{w^{mn}}=0$.

综上所述,所求结果为$\frac{1}{a}\cdot da=d$.
\end{solution}

设$n\geq 2$为正整数, $a,b$为正数.设$\frac{1}{n+1}\left(a^n+a^{n-1}b+\cdots+b^n\right)$与$\left(\frac{a+b}{2}\right)^n$的大小关系.

posted on 2020-09-02 11:33  Eufisky  阅读(1086)  评论(0编辑  收藏  举报

导航